Vanishing of certain coefficients coming from Coxeter groupsDouble quotients of Coxeter groups have the chain property?Centralizers of reflections in special subgroups of Coxeter groupsCross between the nil-Hecke ring and the group ring of a Coxeter groupReference request: Reduced reflection length in Coxeter groupsWhat is the relation between Coxeter transformations of generalized Cartan matrices and Coxeter transformations of finite-dimensional algebras?What is the relation between Coxeter transformations of Coxeter systems and Coxeter transformations of generalized Cartan matrices?A spin extension of a Coxeter group?What are the $2 times 2$ matrix generators of $textSL_2big(mathbbZ[i]big)(2+i)$?Coxeter groups generated by one finite conjugacy classRelation between groups $A_n$, $B_n$, $D_n$ and $S_n$ or inversions of random elements in Coxeter groups

Vanishing of certain coefficients coming from Coxeter groups


Double quotients of Coxeter groups have the chain property?Centralizers of reflections in special subgroups of Coxeter groupsCross between the nil-Hecke ring and the group ring of a Coxeter groupReference request: Reduced reflection length in Coxeter groupsWhat is the relation between Coxeter transformations of generalized Cartan matrices and Coxeter transformations of finite-dimensional algebras?What is the relation between Coxeter transformations of Coxeter systems and Coxeter transformations of generalized Cartan matrices?A spin extension of a Coxeter group?What are the $2 times 2$ matrix generators of $textSL_2big(mathbbZ[i]big)(2+i)$?Coxeter groups generated by one finite conjugacy classRelation between groups $A_n$, $B_n$, $D_n$ and $S_n$ or inversions of random elements in Coxeter groups













6












$begingroup$


Let $left(Wtext, Sright)$ be a Coxeter system. For every $win W$ let us write $left|wright|$ for the length of $w$. Set $lambdaleft(eright)=1$ where $ein W$ denotes the neutral element of the group and define coefficients $lambdaleft(wright)inmathbbZ$ recursively via begineqnarray sum_-leftlambdaleft(vright)=0 endeqnarray for any $win W$. For example this gives us $lambdaleft(eright)=1$, $lambdaleft(sright)=left(-1right)$ for all $sin S$, $lambdaleft(stright)=1$ if $m_st=2$ and $lambdaleft(stright)=0$ if $m_stneq2$ (for the notation see Wikipedia) for all $stext, tin S$ with $sneq t$, and so on.




My question is: Does there always exist some $linmathbbN$ such that $lambdaleft(wright)=0$ for all $win W$ with $left|wright|geq l$?




In the case that $left(Wtext, Sright)$ is right-angled (i.e. $m_stinleft 2text, inftyright$ for all $stext, tin S$ with $sneq t$) this is true and we can take $l=left|Sright|$. I'm wondering if in the general case this property also holds. If no, is it possible to characterize the property in an instructive way?










share|cite|improve this question









$endgroup$
















    6












    $begingroup$


    Let $left(Wtext, Sright)$ be a Coxeter system. For every $win W$ let us write $left|wright|$ for the length of $w$. Set $lambdaleft(eright)=1$ where $ein W$ denotes the neutral element of the group and define coefficients $lambdaleft(wright)inmathbbZ$ recursively via begineqnarray sum_-leftlambdaleft(vright)=0 endeqnarray for any $win W$. For example this gives us $lambdaleft(eright)=1$, $lambdaleft(sright)=left(-1right)$ for all $sin S$, $lambdaleft(stright)=1$ if $m_st=2$ and $lambdaleft(stright)=0$ if $m_stneq2$ (for the notation see Wikipedia) for all $stext, tin S$ with $sneq t$, and so on.




    My question is: Does there always exist some $linmathbbN$ such that $lambdaleft(wright)=0$ for all $win W$ with $left|wright|geq l$?




    In the case that $left(Wtext, Sright)$ is right-angled (i.e. $m_stinleft 2text, inftyright$ for all $stext, tin S$ with $sneq t$) this is true and we can take $l=left|Sright|$. I'm wondering if in the general case this property also holds. If no, is it possible to characterize the property in an instructive way?










    share|cite|improve this question









    $endgroup$














      6












      6








      6





      $begingroup$


      Let $left(Wtext, Sright)$ be a Coxeter system. For every $win W$ let us write $left|wright|$ for the length of $w$. Set $lambdaleft(eright)=1$ where $ein W$ denotes the neutral element of the group and define coefficients $lambdaleft(wright)inmathbbZ$ recursively via begineqnarray sum_-leftlambdaleft(vright)=0 endeqnarray for any $win W$. For example this gives us $lambdaleft(eright)=1$, $lambdaleft(sright)=left(-1right)$ for all $sin S$, $lambdaleft(stright)=1$ if $m_st=2$ and $lambdaleft(stright)=0$ if $m_stneq2$ (for the notation see Wikipedia) for all $stext, tin S$ with $sneq t$, and so on.




      My question is: Does there always exist some $linmathbbN$ such that $lambdaleft(wright)=0$ for all $win W$ with $left|wright|geq l$?




      In the case that $left(Wtext, Sright)$ is right-angled (i.e. $m_stinleft 2text, inftyright$ for all $stext, tin S$ with $sneq t$) this is true and we can take $l=left|Sright|$. I'm wondering if in the general case this property also holds. If no, is it possible to characterize the property in an instructive way?










      share|cite|improve this question









      $endgroup$




      Let $left(Wtext, Sright)$ be a Coxeter system. For every $win W$ let us write $left|wright|$ for the length of $w$. Set $lambdaleft(eright)=1$ where $ein W$ denotes the neutral element of the group and define coefficients $lambdaleft(wright)inmathbbZ$ recursively via begineqnarray sum_-leftlambdaleft(vright)=0 endeqnarray for any $win W$. For example this gives us $lambdaleft(eright)=1$, $lambdaleft(sright)=left(-1right)$ for all $sin S$, $lambdaleft(stright)=1$ if $m_st=2$ and $lambdaleft(stright)=0$ if $m_stneq2$ (for the notation see Wikipedia) for all $stext, tin S$ with $sneq t$, and so on.




      My question is: Does there always exist some $linmathbbN$ such that $lambdaleft(wright)=0$ for all $win W$ with $left|wright|geq l$?




      In the case that $left(Wtext, Sright)$ is right-angled (i.e. $m_stinleft 2text, inftyright$ for all $stext, tin S$ with $sneq t$) this is true and we can take $l=left|Sright|$. I'm wondering if in the general case this property also holds. If no, is it possible to characterize the property in an instructive way?







      gr.group-theory coxeter-groups cayley-graphs






      share|cite|improve this question













      share|cite|improve this question











      share|cite|improve this question




      share|cite|improve this question










      asked Jun 20 at 19:00









      worldreporter14worldreporter14

      2351 silver badge12 bronze badges




      2351 silver badge12 bronze badges




















          1 Answer
          1






          active

          oldest

          votes


















          8












          $begingroup$

          The answer is yes. I claim there are at most $2^n$ elements of $W$ for which $lambda(w) neq 0$. Specifically, let $vee$ be the join in weak order, which is a semi-lattice. If $ s_1, s_2, ldots, s_j subseteq S$ and $s_1 vee s_2 vee cdots vee s_j$ is defined, then I claim that $lambda(s_1 vee s_2 vee cdots vee s_j)=(-1)^j$, and otherwise I claim that $lambda(w)=0$. Thus $N$ can be taken to be the greatest length of $s_1 vee s_2 vee cdots vee s_j$, restricting ourselves to cases where this join is defined.



          The condition $|v^-1 w| = |w| - |v|$ says that $v leq w$ in weak order. Therefore, this is the recursion defining the Mobius function of weak order.
          The Mobius function of a finite lattice $L$ can be computed by Rota's crosscut theorem. (The first online reference I could find was Theorem 1.3 here.) Namely, let $A$ be the set of minimal elements of $L$ -- in this case, this is the set $S$. Then
          $$mu(x) = sum_B subseteq A, bigvee B = x (-1)^.$$



          So $mu(w)=0$ if $w$ is not a join of elements of $S$. If $w$ is a join of elements of $S$, then it is so in only one way (this is a way that weak order is simpler than a general lattice) so we get the description from the first paragraph.






          share|cite|improve this answer











          $endgroup$












          • $begingroup$
            Thanks for your reply. Great answer!
            $endgroup$
            – worldreporter14
            Jun 24 at 9:34













          Your Answer








          StackExchange.ready(function()
          var channelOptions =
          tags: "".split(" "),
          id: "504"
          ;
          initTagRenderer("".split(" "), "".split(" "), channelOptions);

          StackExchange.using("externalEditor", function()
          // Have to fire editor after snippets, if snippets enabled
          if (StackExchange.settings.snippets.snippetsEnabled)
          StackExchange.using("snippets", function()
          createEditor();
          );

          else
          createEditor();

          );

          function createEditor()
          StackExchange.prepareEditor(
          heartbeatType: 'answer',
          autoActivateHeartbeat: false,
          convertImagesToLinks: true,
          noModals: true,
          showLowRepImageUploadWarning: true,
          reputationToPostImages: 10,
          bindNavPrevention: true,
          postfix: "",
          imageUploader:
          brandingHtml: "Powered by u003ca class="icon-imgur-white" href="https://imgur.com/"u003eu003c/au003e",
          contentPolicyHtml: "User contributions licensed under u003ca href="https://creativecommons.org/licenses/by-sa/3.0/"u003ecc by-sa 3.0 with attribution requiredu003c/au003e u003ca href="https://stackoverflow.com/legal/content-policy"u003e(content policy)u003c/au003e",
          allowUrls: true
          ,
          noCode: true, onDemand: true,
          discardSelector: ".discard-answer"
          ,immediatelyShowMarkdownHelp:true
          );



          );













          draft saved

          draft discarded


















          StackExchange.ready(
          function ()
          StackExchange.openid.initPostLogin('.new-post-login', 'https%3a%2f%2fmathoverflow.net%2fquestions%2f334448%2fvanishing-of-certain-coefficients-coming-from-coxeter-groups%23new-answer', 'question_page');

          );

          Post as a guest















          Required, but never shown

























          1 Answer
          1






          active

          oldest

          votes








          1 Answer
          1






          active

          oldest

          votes









          active

          oldest

          votes






          active

          oldest

          votes









          8












          $begingroup$

          The answer is yes. I claim there are at most $2^n$ elements of $W$ for which $lambda(w) neq 0$. Specifically, let $vee$ be the join in weak order, which is a semi-lattice. If $ s_1, s_2, ldots, s_j subseteq S$ and $s_1 vee s_2 vee cdots vee s_j$ is defined, then I claim that $lambda(s_1 vee s_2 vee cdots vee s_j)=(-1)^j$, and otherwise I claim that $lambda(w)=0$. Thus $N$ can be taken to be the greatest length of $s_1 vee s_2 vee cdots vee s_j$, restricting ourselves to cases where this join is defined.



          The condition $|v^-1 w| = |w| - |v|$ says that $v leq w$ in weak order. Therefore, this is the recursion defining the Mobius function of weak order.
          The Mobius function of a finite lattice $L$ can be computed by Rota's crosscut theorem. (The first online reference I could find was Theorem 1.3 here.) Namely, let $A$ be the set of minimal elements of $L$ -- in this case, this is the set $S$. Then
          $$mu(x) = sum_B subseteq A, bigvee B = x (-1)^.$$



          So $mu(w)=0$ if $w$ is not a join of elements of $S$. If $w$ is a join of elements of $S$, then it is so in only one way (this is a way that weak order is simpler than a general lattice) so we get the description from the first paragraph.






          share|cite|improve this answer











          $endgroup$












          • $begingroup$
            Thanks for your reply. Great answer!
            $endgroup$
            – worldreporter14
            Jun 24 at 9:34















          8












          $begingroup$

          The answer is yes. I claim there are at most $2^n$ elements of $W$ for which $lambda(w) neq 0$. Specifically, let $vee$ be the join in weak order, which is a semi-lattice. If $ s_1, s_2, ldots, s_j subseteq S$ and $s_1 vee s_2 vee cdots vee s_j$ is defined, then I claim that $lambda(s_1 vee s_2 vee cdots vee s_j)=(-1)^j$, and otherwise I claim that $lambda(w)=0$. Thus $N$ can be taken to be the greatest length of $s_1 vee s_2 vee cdots vee s_j$, restricting ourselves to cases where this join is defined.



          The condition $|v^-1 w| = |w| - |v|$ says that $v leq w$ in weak order. Therefore, this is the recursion defining the Mobius function of weak order.
          The Mobius function of a finite lattice $L$ can be computed by Rota's crosscut theorem. (The first online reference I could find was Theorem 1.3 here.) Namely, let $A$ be the set of minimal elements of $L$ -- in this case, this is the set $S$. Then
          $$mu(x) = sum_B subseteq A, bigvee B = x (-1)^.$$



          So $mu(w)=0$ if $w$ is not a join of elements of $S$. If $w$ is a join of elements of $S$, then it is so in only one way (this is a way that weak order is simpler than a general lattice) so we get the description from the first paragraph.






          share|cite|improve this answer











          $endgroup$












          • $begingroup$
            Thanks for your reply. Great answer!
            $endgroup$
            – worldreporter14
            Jun 24 at 9:34













          8












          8








          8





          $begingroup$

          The answer is yes. I claim there are at most $2^n$ elements of $W$ for which $lambda(w) neq 0$. Specifically, let $vee$ be the join in weak order, which is a semi-lattice. If $ s_1, s_2, ldots, s_j subseteq S$ and $s_1 vee s_2 vee cdots vee s_j$ is defined, then I claim that $lambda(s_1 vee s_2 vee cdots vee s_j)=(-1)^j$, and otherwise I claim that $lambda(w)=0$. Thus $N$ can be taken to be the greatest length of $s_1 vee s_2 vee cdots vee s_j$, restricting ourselves to cases where this join is defined.



          The condition $|v^-1 w| = |w| - |v|$ says that $v leq w$ in weak order. Therefore, this is the recursion defining the Mobius function of weak order.
          The Mobius function of a finite lattice $L$ can be computed by Rota's crosscut theorem. (The first online reference I could find was Theorem 1.3 here.) Namely, let $A$ be the set of minimal elements of $L$ -- in this case, this is the set $S$. Then
          $$mu(x) = sum_B subseteq A, bigvee B = x (-1)^.$$



          So $mu(w)=0$ if $w$ is not a join of elements of $S$. If $w$ is a join of elements of $S$, then it is so in only one way (this is a way that weak order is simpler than a general lattice) so we get the description from the first paragraph.






          share|cite|improve this answer











          $endgroup$



          The answer is yes. I claim there are at most $2^n$ elements of $W$ for which $lambda(w) neq 0$. Specifically, let $vee$ be the join in weak order, which is a semi-lattice. If $ s_1, s_2, ldots, s_j subseteq S$ and $s_1 vee s_2 vee cdots vee s_j$ is defined, then I claim that $lambda(s_1 vee s_2 vee cdots vee s_j)=(-1)^j$, and otherwise I claim that $lambda(w)=0$. Thus $N$ can be taken to be the greatest length of $s_1 vee s_2 vee cdots vee s_j$, restricting ourselves to cases where this join is defined.



          The condition $|v^-1 w| = |w| - |v|$ says that $v leq w$ in weak order. Therefore, this is the recursion defining the Mobius function of weak order.
          The Mobius function of a finite lattice $L$ can be computed by Rota's crosscut theorem. (The first online reference I could find was Theorem 1.3 here.) Namely, let $A$ be the set of minimal elements of $L$ -- in this case, this is the set $S$. Then
          $$mu(x) = sum_B subseteq A, bigvee B = x (-1)^.$$



          So $mu(w)=0$ if $w$ is not a join of elements of $S$. If $w$ is a join of elements of $S$, then it is so in only one way (this is a way that weak order is simpler than a general lattice) so we get the description from the first paragraph.







          share|cite|improve this answer














          share|cite|improve this answer



          share|cite|improve this answer








          edited Jun 20 at 23:20

























          answered Jun 20 at 19:32









          David E SpeyerDavid E Speyer

          109k10 gold badges289 silver badges554 bronze badges




          109k10 gold badges289 silver badges554 bronze badges











          • $begingroup$
            Thanks for your reply. Great answer!
            $endgroup$
            – worldreporter14
            Jun 24 at 9:34
















          • $begingroup$
            Thanks for your reply. Great answer!
            $endgroup$
            – worldreporter14
            Jun 24 at 9:34















          $begingroup$
          Thanks for your reply. Great answer!
          $endgroup$
          – worldreporter14
          Jun 24 at 9:34




          $begingroup$
          Thanks for your reply. Great answer!
          $endgroup$
          – worldreporter14
          Jun 24 at 9:34

















          draft saved

          draft discarded
















































          Thanks for contributing an answer to MathOverflow!


          • Please be sure to answer the question. Provide details and share your research!

          But avoid


          • Asking for help, clarification, or responding to other answers.

          • Making statements based on opinion; back them up with references or personal experience.

          Use MathJax to format equations. MathJax reference.


          To learn more, see our tips on writing great answers.




          draft saved


          draft discarded














          StackExchange.ready(
          function ()
          StackExchange.openid.initPostLogin('.new-post-login', 'https%3a%2f%2fmathoverflow.net%2fquestions%2f334448%2fvanishing-of-certain-coefficients-coming-from-coxeter-groups%23new-answer', 'question_page');

          );

          Post as a guest















          Required, but never shown





















































          Required, but never shown














          Required, but never shown












          Required, but never shown







          Required, but never shown

































          Required, but never shown














          Required, but never shown












          Required, but never shown







          Required, but never shown







          Popular posts from this blog

          Get product attribute by attribute group code in magento 2get product attribute by product attribute group in magento 2Magento 2 Log Bundle Product Data in List Page?How to get all product attribute of a attribute group of Default attribute set?Magento 2.1 Create a filter in the product grid by new attributeMagento 2 : Get Product Attribute values By GroupMagento 2 How to get all existing values for one attributeMagento 2 get custom attribute of a single product inside a pluginMagento 2.3 How to get all the Multi Source Inventory (MSI) locations collection in custom module?Magento2: how to develop rest API to get new productsGet product attribute by attribute group code ( [attribute_group_code] ) in magento 2

          Category:9 (number) SubcategoriesMedia in category "9 (number)"Navigation menuUpload mediaGND ID: 4485639-8Library of Congress authority ID: sh85091979ReasonatorScholiaStatistics

          Magento 2.3: How do i solve this, Not registered handle, on custom form?How can i rewrite TierPrice Block in Magento2magento 2 captcha not rendering if I override layout xmlmain.CRITICAL: Plugin class doesn't existMagento 2 : Problem while adding custom button order view page?Magento 2.2.5: Overriding Admin Controller sales/orderMagento 2.2.5: Add, Update and Delete existing products Custom OptionsMagento 2.3 : File Upload issue in UI Component FormMagento2 Not registered handleHow to configured Form Builder Js in my custom magento 2.3.0 module?Magento 2.3. How to create image upload field in an admin form